10

Let $m$ and $n$ be two nonnegative integers. Assume that there is a group isomorphism $\mathbb{Z}^m \cong \mathbb{Z}^n$. Prove that $m = n$.

I tried using a contrapositive, ($m \neq n$ implies $\mathbb{Z}^m \ncong \mathbb{Z}^n$), and I think the problem is that there won't be a homomorphism, but did not get anywhere. Is there a better approach to this problem?

Shaun
  • 44,997
chubbycantorset
  • 2,065
  • 1
  • 20
  • 33

6 Answers6

15

Once again, I'll give you the outline, you fill in the details.

Suppose that $\mathbb{Z}^m\cong\mathbb{Z}^n$. Then, you see that $\mathbb{Z}^m\otimes_{\mathbb{Z}}\mathbb{Z}_2\cong\mathbb{Z}^n\otimes_{\mathbb{Z}}\mathbb{Z}_2$, which tells you that $\mathbb{Z}_2^m\cong\mathbb{Z}_2^n$, and so $2^m=2^n$--thus $m=n$.

EDIT: Since you don't know tensor products, perhaps this will be more understandable. If $\mathbb{Z}^m\cong\mathbb{Z}^n$, then $\text{Hom}_\mathbb{Z}(\mathbb{Z}^m,\mathbb{Q})\cong\text{Hom}_\mathbb{Z}(\mathbb{Z}^n,\mathbb{Q})$ as $\mathbb{Q}$-vector spaces. But, you can prove that as $\mathbb{Q}$-vector spaces one has that $\text{Hom}_\mathbb{Z}(\mathbb{Z}^k,\mathbb{Q})\cong\mathbb{Q}^k$, and so we have $\mathbb{Q}^m\cong\mathbb{Q}^n$, from where normal vector space theory tells us that $m=n$.

EDIT EDIT: You seemed to only take issue with the previous proof because of the vector spaces. In patticular,you seemed ok with the Hom manipulation. Here's a way to combine the first and the previous proof! Show first that $\text{Hom}(\mathbb{Z}^k,A)\cong A^k$ for any abelian group A. Then apply this fact to show that our problem implies $\mathbb{Z}_2^m\cong\mathbb{Z}_2^n$ again.

Alex Youcis
  • 54,059
  • Can you explain what $\otimes_{\mathbb{Z}}$ means? Is it just the direct product? I've not seen this notation before. – chubbycantorset Dec 09 '12 at 07:42
  • 1
    It's the tensor product. Look at my edit, now that I see you don't know tensor products. – Alex Youcis Dec 09 '12 at 07:44
  • Alternatively, couldn't you go from the group isomorphism to a module isomorphism, and then use the fact that the rank is invariant? – wj32 Dec 09 '12 at 07:46
  • @wj32 This is, in essence, what he's trying to prove. I doubt he would be given this problem if he knew that $\mathbb{Z}$ had the IBN property--then this would literally be the problem! – Alex Youcis Dec 09 '12 at 07:47
  • @AlexYoucis: Oh yes, you're right. – wj32 Dec 09 '12 at 07:48
  • No idea what a module is, or what invariant means. Also, @AlexYoucis: We haven't been taught anything about vector spaces, and hence aren't allowed to use results from it on our exams. Is it possible to only allude to group theoretic concepts? Btw, for this question, maybe it would suffice to prove that if $A \cong B$ then $A/pA \cong B/pB$, where $p \in \mathbb{N}$. I think that's essentially what you're saying. – chubbycantorset Dec 09 '12 at 07:51
  • The problem with what you've said is that you can't conclude that $A/pA\cong B/pB$, can you? You can either proceed as Brian Scott mentions, or you can do this by just showing that an isomorphism $\mathbb{Z}^m\xrightarrow{\simeq}\mathbb{Z}^n$ induces an isomorphism $\mathbb{Z}_2^m\xrightarrow{\simeq}\mathbb{Z}_2^n$ in a somewhat natural way. There is a hint to this in Dummit and Foote I believe. – Alex Youcis Dec 09 '12 at 07:54
  • @chubbycantorset Pushing all the tensor products away the main idea of the proof is that we convert things about free modules (whatever they are) into things about vector spaces where we can calculate dimension. That is the essence of the proof above. –  Dec 09 '12 at 08:08
  • Dear @AlexYoucis: I don't want to post this as a separate answer because it is essentially similar in nature to yours, but could you also include the argument by localization, which gives us $\mathbb{Q}^m \cong \mathbb{Q}^n$ in your answer? – Rankeya Dec 09 '12 at 08:09
  • 1
    @Rankeya Sure, localization commutes with products. But really, localizing at, say $0$, is the same thing as tensoring with $\mathbb{Q}$. – Alex Youcis Dec 09 '12 at 08:11
  • Agreed. But one can frame it in a way that tensor products are avoided altogether. – Rankeya Dec 09 '12 at 08:14
  • @Rankeya True, but I think that with the level of sophistication needed to talk about localization (and to do the proof I have in mind you need at least a little bit of sophistication) it's likely that anyone not able to use tensor products, won't be able to use localizations. – Alex Youcis Dec 09 '12 at 08:15
  • @AlexYoucis and Rankeya, I think that localization and tensor products are at the same level of sophistication for this problem. –  Dec 09 '12 at 08:24
  • @BenjaLim You can actually prove, using the same technique and a fact about exterior powers of free modules, that there can't exist injective maps $\mathbb{Z}^m\to\mathbb{Z}^n$, $m>n$, or surjective maps $\mathbb{Z}^m\to\mathbb{Z}^n$, $n>m$. But, the points all moot because that's way overkill. – Alex Youcis Dec 09 '12 at 08:37
  • @AlexYoucis I deleted my comment on exterior algebras because it wouldn't be helpful to the OP. I will post an alternative answer soon that will hopefully rely on less "highfalutin" methods. I do not mean to imply anything negative with that word (in fact your way was the first proof that came to mind in this problem) but unfortunately for a beginner a high level proof may not be helpful. For example I posted an answer here: http://math.stackexchange.com/questions/211481/the-determinant-function-is-the-only-one-satisfying-the-conditions/211489#211489 that was not helpful at all to the OP. –  Dec 09 '12 at 08:37
  • @AlexYoucis I have posted an answer below. –  Dec 09 '12 at 10:28
  • @AlexYoucis I just realised in your proof above you could have tensored over $\Bbb{Z}$ with any field and the proof would have gone through. –  Dec 09 '12 at 12:00
  • 1
    @BenjaLim Obviously this was true. But, tensoring with a finite quotient field allows one to bypass having to prove the result for fields. – Alex Youcis Dec 09 '12 at 16:30
  • @chubbycantorset Do you know that if you have two vector spaces say $\Bbb{R}^n$ and $\Bbb{R}^m$ that are isomorphic then $m = n$? Hint: Rank - Nullity Theorem. –  Dec 09 '12 at 23:25
  • @BenjaLim The OP explicitly said that they cannot use facts about vector spaces--and I think that proving the well-definedness of dimension for vector spaces is simpler than applying the Rank Nullity Theorem. – Alex Youcis Dec 09 '12 at 23:43
  • @AlexYoucis I Just realised something in your proof above. If we have an isomorphism $f : \Bbb{Z}^m \to \Bbb{Z}^n$ then why should the tensored map $f \otimes 1$ be injective? As an example we consider $0 \to \Bbb{Z} \stackrel{2.}{\to} \Bbb{Z}$. Then after tensoring over $\Bbb{Z}$ with $\Bbb{Z}/2\Bbb{Z}$ we get that $f\otimes 1$ is actually the zero map! I think you can only conclude that after tensoring you have a *surjection* between $\Bbb{Z}^m \otimes_\Bbb{Z} \Bbb{Z}/2$ and $\Bbb{Z}^n \otimes_\Bbb{Z} \Bbb{Z}/2$ no? –  Dec 10 '12 at 12:43
  • @AlexYoucis However over a PID (like $\Bbb{Z}$ which you are tensoring over) we recall that a $\Bbb{Z}$ - module is flat iff it is torsion free. $\Bbb{Z}/2\Bbb{Z}$ is a finite abelian group and hence is not flat. On the other hand $\Bbb{Q}$ is torsion free and so is flat. Hence I think in the proof above for it to go through we need to tensor over $\Bbb{Q}$. –  Dec 10 '12 at 12:56
  • @BenjaLim Tensoring with a fixed entry, such as $-\otimes_\mathbb{Z}\mathbb{Z}_2$ is a functor--it takes isomorphisms to isomorphisms. While, in general, it is true that the tensor functor is exact (this is what flatness is) we only need its functorial properties here. – Alex Youcis Dec 10 '12 at 13:05
  • @AlexYoucis Hmmmm I am confused now I can see that a functor takes isomorphisms to isomorphisms, but perhaps I'm stupid I am seeing that it takes injections to injections. –  Dec 10 '12 at 13:19
  • @BenjaLim It does not, it takes injections to injections precisely when the fixed module is flat, but that is not important here. Every single functor is isomorphism preserving, only special ones are exact. So, in particular while functors preserve properties that have to do with mappings and identities isomorphism (there exists an inverse), having a section or a retraction (having a left or right inverse), properties not phrased this way such as injectivity aren't in general preserved. If you have more questions you can email me, but we should stop this convo. – Alex Youcis Dec 10 '12 at 13:41
12

Let $G=\mathbb{Z}^n$ and $H=\mathbb{Z}^m$. Then $G/2G = (\mathbb{Z}/2\mathbb{Z})^n$. In particular, $|G/2G| = 2^n$. By a similar argument $|H/2H| = 2^m$. If they are isomorphic then $2^n = 2^m$.

5

You can prove it in the same way that you prove that the dimension of a vector space is well-defined.

Suppose that $m\le n$ and $h:\Bbb Z^m\to\Bbb Z^n$ is an isomorphism. For $k=1,\dots,m$ let $e_k$ be the element $\langle a_1,\dots,a_m\rangle$ such that $a_k=1$ and $a_i=0$ for $i\ne k$. Note for any $\langle a_1,\dots,a_m\rangle\in\Bbb Z^m$,

$$\langle a_1,\dots,a_m\rangle=\sum_{k=1}^ma_ke_k\;,$$

and this representation is unique: if

$$\sum_{k=1}^ma_ke_k=\sum_{k=1}^mb_ke_k\;,$$ then $$\sum_{k=1}^m(a_k-b_k)e_k=\langle \underbrace{0,\dots,0}_m\rangle\;,$$ and therefore $a_1=b_1,\dots,a_m=b_m$. In other words, $\{e_1,\dots,e_m\}$ behaves very much like a basis for a vector space.

Now show that $\{h(e_1),\dots,h(e_n)\}$ behaves like a basis for $\Bbb Z^n$: each $z\in\Bbb Z^n$ can be written uniquely in the form $$z=\sum_{k=1}^ma_kh(e_k)$$ for some integers $a_k$, $k=1,\dots,m$.

Now get a contradiction when $n>m$ by considering the elements of $\Bbb Z^n$ that are analogous to the $e_k\in\Bbb Z^m$.

user26857
  • 52,094
Brian M. Scott
  • 616,228
  • This makes perfect sense, but as I just said in reply to Alex's comment, we aren't allowed to use anything related to vector spaces in our exams. – chubbycantorset Dec 09 '12 at 07:55
  • 2
    @chubbycantorset: Well $\mathbb{Z}^n$ isn't exactly a vector space here, and you're not using any existing results about vector spaces in this proof. – wj32 Dec 09 '12 at 07:57
  • 4
    @chubbycantorset: Aargh! In my opinion it’s a poor instructor who can’t come up with good problems without introducing stupid artificial restrictions $-$ and if that prohibition covers this argument, it qualifies as stupid. (Sorry for the outburst, but that’s been a pet peeve of mine for decades.) – Brian M. Scott Dec 09 '12 at 07:57
  • I certainly agree. We've been shown pretty much no applications of theorems, and hence I find it more difficult to manipulate n-cycles or computing orders of groups than proving results that follow theorems. Incidentally, would you know any online resources that show you how to apply group theoretical concepts? – chubbycantorset Dec 09 '12 at 08:02
  • @chubbycantorset: One fun application is in combinatorics. Download this book and take a look at Chapter 6. Also, this book is good: Applied Abstract Algebra – wj32 Dec 09 '12 at 08:07
  • @chubbycantorset: I don’t, I’m afraid; I’m not really an algebraist, and I’ve never actually gone looking. – Brian M. Scott Dec 09 '12 at 08:07
  • @chubbycantorset: Just purely for your amusement, you can use the notion of quasi-isometry to come up with yet another proof of this statement. Indeed this was one of the first things I learned in a course on geometric group theory. – Rankeya Dec 09 '12 at 08:19
  • @Ryneka What is that way? I am unfamiliar. – Alex Youcis Dec 09 '12 at 09:53
  • 1
    In such a direct approach this is actually the question: how to "get a contradiction when $n>m$ by considering the elements of $\Bbb Z^n$ that are analogous to the $e_k\in\Bbb Z^m$." The rest of your answer is about well known things which eventually could help, but nothing more. (For short, this answer is still far from a proof.) – user26857 Feb 08 '15 at 09:39
  • @user26857: No. The point of the answer is the point of view involved, and carrying out the final step is straightforward and completes the argument. And of course it's not a proof; it doesn't purport to be one. It's an extended hint. – Brian M. Scott Feb 08 '15 at 09:46
  • I have no idea how one can prove in a straightforward manner the last step. (In fact, this is all about the rank of a finitely generated free module, and if I remember well this was never something trivial, or straightforward if you like it.) – user26857 Feb 08 '15 at 09:50
  • @user26857: Pretty much the same way you’d prove it for $\Bbb Q^m$ and $\Bbb Q^n$. Either of the arguments here works with minor modifications, since elements of $\Bbb Z^n$ are linearly independent over $\Bbb Z$ iff they’re lin. indep. over $\Bbb Q$. – Brian M. Scott Feb 08 '15 at 20:17
3

Highfalutin approach: Suppose we have an isomorphism $f : \Bbb{Z}^m \stackrel{\simeq}{\longrightarrow} \Bbb{Z}^n$. Then we recall that a module over a PID is flat iff it is torsion-free. Considering $\Bbb{Q}$ as an abelian group, it will now follow that the functor $- \otimes_{\Bbb{Z}} \Bbb{Q}$ is an exact functor and so

$$f \otimes 1 : \Bbb{Z}^m \otimes_{\Bbb{Z}} \Bbb{Q} \to \Bbb{Z}^n \otimes_{\Bbb{Z}} \Bbb{Q}$$

is injective. It is also clearly surjective and hence is an isomorphism. Using the fact that

  1. Tensor products commute with direct sums
  2. For any $R$ - module $M$ we have a canonical isomorphism $R \otimes_R M\cong M$ which on elementary tensors sends $r \otimes m \to rm$

we conclude that $f \otimes 1$ is an isomorphism between $\Bbb{Q}^m$ and $\Bbb{Q}^n$ and rank - nullity now gives that $m = n$.

More concrete approach:

Let us try to understand via elementary methods why there can be no isomorphism between say $\Bbb{Z}^3$ and $\Bbb{Z}^2$. Suppose there were some isomorphism $f : \Bbb{Z}^2 \to \Bbb{Z}^3$. Then if $x_1,x_2,x_3$ are the canonical basis generators for $\Bbb{Z}^3$ and $y_1,y_2$ that of $\Bbb{Z}^2$ we can find integers $a_{11},a_{12},a_{21},a_{22},a_{31},a_{32}$ such that

$$\sum_{j=1}^2 a_{ij}y_j = x_i$$

for $1 \leq j \leq 3$. More concretely, this means that given any triple $(a,b,c)$ we can find integers $d,e$ such that

$$\begin{pmatrix} a_{11} & a_{12} \\ a_{21} & a_{22} \\ a_{31} & a_{32} \end{pmatrix} \begin{pmatrix} d \\ e \end{pmatrix} = \begin{pmatrix} a \\ b \\ c \end{pmatrix}.$$

Now from elementary row reduction when working over a field we know that this is not going to be possible simply because the number of pivots is only going to be $\leq 2$ and not $3$. But what if we work over the integers? I will now show using a concrete example that we can always get the last row to be zero. This will then give a contradiction because the matrix applied to no pair $(d,e)$ of integers will ever be equal to

$$\begin{pmatrix} 0 \\ 0 \\ 1\end{pmatrix}.$$

Now for a concrete example. Suppose you take the matrix $$\begin{pmatrix} 7 & 6 \\ 5 & 12 \\ 4 & 1 \end{pmatrix}.$$

Let $R_1$ mean row $1$, $R_2$ row $2$ and so on. Then if we do $7R_3 - 4R_1$ and $7R_2 - 5R_1$ we get the matrix

$$\begin{pmatrix} 7 & 6 \\ 0 & 54 \\ 0 & -17 \end{pmatrix}.$$

But now if we do $54R_3 + 17R_2$ the last row will be zero and you get your desired contradiction.

From this concrete example do you see now why there can never be an isomorphism between $\Bbb{Z}^2$ and $\Bbb{Z}^3$. More generally do you see why there can never be an isomorphism between $\Bbb{Z}^m$ and $\Bbb{Z}^n$ for $m\neq n$?

3

Embed $\Bbb Z$ in $\Bbb Q$ and thereby $\Bbb Z^n$ in $\Bbb Q^n$. Now clearly $\Bbb Z^n$ has an $n$-tuple of elements that are linearly independent over $\Bbb Z$ (the standard basis will do as an example), and it does not have any $(n+1)$-tuple that is linearly independent over $\Bbb Z$, since already $\Bbb Q^n$ doesn't admit any $(n+1)$-tuple that is linearly independent over $\Bbb Q$, let alone over $\Bbb Z$. Then you can recover the $n$ in $A\cong\Bbb Z^n$ as the maximum number of linearly independent elements over $\Bbb Z$ one can find in the free Abelian group $A$. Thus $\Bbb Z^m\cong\Bbb Z^n$ implies $m=n$.

0

Suppose $m\neq n$,

$$\Bbb Z^m\cong\langle x_1,\dots, x_m\mid \{x_ix_j=x_jx_i: i,j\in\{1,\dots m\}\}\rangle,$$

and

$$\Bbb Z^n\cong\langle y_1,\dots, y_n\mid \{y_ry_s=y_sy_r: r,s\in\{1,\dots n\}\}\rangle.$$

In each presentation, kill $z^2$ for each element $z$. Then we get, by abelianness,

$$G:=\Bbb Z^m/\langle \{x_i^2:i\in\{1,\dots, m\} \rangle \cong\langle x_1,\dots, x_m\mid \{x_ix_j=x_jx_i: i,j\in\{1,\dots m\}\}\cup\{x_i^2: i\in\{1,\dots, m\}\}\rangle$$

and

$$H:=\Bbb Z^n/\langle \{y_r^2:r\in\{1,\dots, n\} \rangle \cong\langle y_1,\dots, y_n\mid \{y_ry_s=y_sy_r: i,j\in\{1,\dots n\}\}\cup\{y_r^2: r\in\{1,\dots, n\}\}\rangle.$$

But then

$$G\cong\Bbb Z_2^m\not\cong \Bbb Z_2^n\cong H$$

by this standard result, since $m\neq n$; but we did the same thing to each presentation and yet we got different results. Thus

$$\Bbb Z^m\not\cong\Bbb Z^n.$$

This is the contrapositive of the result in question, so we are done.

Shaun
  • 44,997